Sunteți pe pagina 1din 14

MECH 5403 F Advanced Thermodynamics 1. (20 marks). See Callen (1960), Question 1.9-1 or Callen (1985), Question 1.

10-1 The following equations are purported to be fundamental equations of various thermodynamic systems. Use the information provided in Callen to determine if any of the equations below are physically permissible and sketch the shape of the curve S = S (U) assuming that both the volume V and the number of moles N are constants. (A)

(B)

(C) Quantities and R are positive constants and symbols N, V and U denote the total mole number, volume and internal energy. In all cases where fractional exponents appear only the real, positive root is to be taken as physically acceptable. Demonstrate if any of these expressions is also monotonically increasing? Solutions The requirements for a suitable fundamental equation (in the entropic form) as summarized by Callen can be stated as follows: Postulate I: There exist particular states (called equilibrium states) of simple systems that, macroscopically, are characterized completely by the internal energy U, the total system volume V, and the mole numbers N1, N2, ... , Nr of the chemical components. This postulate is satisfied already in the sense that we have been given functional relationships. Postulate II: There exists a function (called the entropy and denoted by the symbol S) of the extensive parameters of any composite system, defined for all equilibrium states and having the following property - The values assumed by the extensive parameters in the absence of an internal constraint are those that maximize the entropy over the manifold of constrained equilibrium states. Postulate III: The entropy of a composite system is additive over the constituent subsystems. Also, the entropy is continuous and differentiable and it is a monotonically increasing function of the energy. This postulate contains the following mathematical tests that any suitable fundamental eq must satisfy; namely, (1) The entropy must be a continuous, differentiable and monotonically increasing function

of the internal energy so that, for a single component system, we require and ,

(2) The entropy must be a homogeneous, first-order function of its extensive parameters so that where is a constant. Postulate IV (Nernst postulate): The entropy of any system vanishes in the limit of the system going to a state of absolute zero temperature. Mathematically, we require (3) The entropy must be bounded in the limit of low temperature so that as .

(A) Given the proposed function

(1) provided , and

which does not satisfy the monotonically increasing condition. (2)

(3) possible because

can only be achieved if either

or

would require that . Similarly, requiring t also leads to unacceptable conclusion that this can only be achieved if . Thus, this equation is not an acceptable fundamental equation because it does not satisfy requirement (1b) and (3). Finally, a sketch of S ( U ) versus U would show a simple ln ( U ) dependence.

(B) Given

(1)

if only if

which represents a limited range of validity and thus this propose function is not suitable as a fundamental eq. (2) (3) than or if

will occur if e faster than where we see that as as

constant V and N we see that constants which means that as Also,

in a suitable fashion . since

the result that the entropy as the temperature . Finally, the equation in this part is not a suitable fundamental eq since it does not satisfy the first two conditions (1) and (2). Finally, a sketch of S ( U ) would show the entropy S varying with dependence on the internal energy U as (C) Given .

(1) does not satisfy the first requirement. (2) (3) region. Thus, this proposed relation is not suitable as it violates all conditions.

for

as

2. (10 marks). The free electrons responsible for conduction in a metal can be regarded as an exotic kind of ideal gas. In this case the equation of state is determined by Tf, a constant, called the Fermi temperature, given by , where Np is the number of free electrons in a volume V, m is the electron mass, h is Plancks constant and k is Boltzmanns constant. When the actual temperature T << Tf , the gas is said to be a degenerate Fermi gas and, to a good approximation, the equation of state is then given by . (a) Metallic sodium has one free electron per atom and the density of sodium is 970 kg/m3 at room temperature. The molar mass is 23 g/mol. Determine Tf. (b) Calculate the pressure of the free electron gas in metallic sodium, in Pascals, at room temperature. Comment on the magnitude of the result. For example, is the pressure large or small compared with an industrial cylinder of compressed gas at say 100 bar? Solutions The number of moles N in a gas is given by the relation

where Np represents the number of particles (atoms or molecules) in the gas, NA is Avogadros number (in particles or molecules per mole; 6.0221 1023 mol-1), M is the mass of the gas considered (in kg) and MW is the molecular weight (in kg mol-1). As a consequence of these relations, the ideal gas equation of state can be written as where R is the molar gas constant, 8.3145 J K-1 mol-1, or it can be written as

where is Boltzmanns constant. (a) One uses the first relation above ,

1.3807 10-23 J K-1

, to obtain the particle density as

where we have used sodiums density and the molar mass in kg mol-1 and the result has units of particles (in this case electrons) per unit volume (units of m3). Inserting this value in the expression for the Fermi temperature yields

indicating that the electrons within a metal are in an incredible, agitated state of motion. The reason for this behaviour is complex and related to the Pauli exclusion principle. U. Mizutami, Introduction to the Electron Theory of Metals, Cambridge Univ. Pr., Cambridge (2001), pg. 26 and hyperphysics.phy-astr.gsu.edu/hbase/tables/fermi.html (b) The pressure of the free electron gas within the sodium metal is estimated from

. which indicates that the electron gas in under enormous pressure. 3. (15 marks). The figure below shows five states of a fixed amount of an ideal, monatomic gas, for which the specific heat . For the states a and b: Va = 2 m3, Pa = 103 Pa, Ta = 300 K, Vb = 1 m3, and Pb = 104 Pa. These states are linked by three different quasi-static processes. The purpose of this exercise is to calculuate Sb - Sa by evaluating the ratio dQ / T along these three paths to confirm that the numerical value obtained in each case agrees with

The process segments are defined as: a6c6e6d is an adiabatic, quasi-static path; d6b is an isothermal process; e6b is a constant pressure heating process; and c6b is a constant volume heating process. Throughout your calculations set Sa = 0.

4. (10 marks). See Callen (1960), Question 2.2-6 or Callen (1985), Question 2.2-6 Find the three equations of state for a system with the fundamental equation

where the symbols vo, and R denote positive, real valued constants. Solution The three equations of state can be obtained from the fundamental eq provided. Thus,

where the symbol

denotes a variation of the internal energy while holding both the volume

and the mole number fixed. The temperature is then obtained as a limiting ratio or

Of course, this expression may also be obtained by more conventional calculus techniques but the variational technique of differentiation is helpful when the expressions are complex. The pressure and chemical potential are obtained directly as

so that

or

, and

where this final expression for the chemical potential can be written as either or 5. (10 marks). See Callen (1985), Question 2.2-7 A particular system is characterized by the fundamental equation where A is an arbitrary constant. If N moles of substance, initially at temperature To and pressure Po, is expanded isentropically (at fixed entropy; s = a constant) until the pressure is halved, then what will be the final temperature? Solution We begin with the given fundamental eq and determine the temperature and pressure equations of state as follows: .

or

or

As stated in the question, PI = Po and TI = To (the initial conditions) while 2PF = PI and we are asked to determine TF when the volume V is expanded isentropically (at fixed entropy S ). Thus,

and

where

which means that 6. (10 marks). Calculate the work and energy input via heat when 2 kg of water undergoes an isothermal process in which the volume changes quasi-statically from 200 m3 to 0.2 m3 at 100oC. Solution From any table or graph for the properties of water and steam is should be apparent that the initial state os om tje vapour region while the final state is in the mixed two-phase region. We consider the process in two steps; denoted by below by symbols (1) and (2). The specific volume of saturated vapour is 1.673 m3 kg-1 at 100oC.

(1) For isothermal compression in the vapour phase we have: VI = 200 m3; VF = 2 1.7 m3 = 3.4 m3. Also, for the vapour phase compression in this case the specific internal energy u is approximately constant, and for the purposes of this calculation we shall treat the vapour as an ideal gas. Now, from the tabulated or graphical representation of water and steam, we have that P = 1.0 105 Pa when the specific volume v = 1.7 m3 kg-1 at 100oC. Hence for 2 kg of water vapour at 100oC, P V = 1.01 105 1.673 2 = 3.38 105 J and thus the work required is

and since

we have that

(2) For isothermal, isobaric (constant pressure) compression within the two-phase region we have: VI = 3.4 m3; VF = 0.2 m3 while P = 1.01 105 Pa. The work performed is Now, UI = 2 2.51 MJ and UF = 2 0.55 MJ so the difference is heat outflow for this process is and 7. (10 marks). A vessel with a volume of 15 m3 contains liquid water and steam in equilibrium at 50oC. The mass of water in the liquid phase is 1.0 kg. Determine the mass of water in the vapour phase.

. Henc

. Finally, for the complete process, one obta

Solution Let m denote the mass of vapour phase steam. We know the volume is an extensive variable and we shall assume that the total volume is the sum of the volumes of the individual phases, whether they are mixed or not. From either tabulated or graphical water-steam data we find that the specific volume of saturated water at 50oC is 0.001 m3 kg-1 while, for saturated steam the corresponding value is 12.03 m3 kg-1. Hence, the total volume is given by V = 0.00101 1.0 + 12.03 m = 15 m3. Thus, m ~ 1.25 kg. Please notice that the mass of steam is not very sensitive to the mass of liquid water in the vessel because the volume occupied by the liquid phase is only a small fraction of the vessel. Not until the mass of water is of the order of several hundred kilograms or more will the volume and hence the mass of steam in the vessel become significantly affected. 8. (15 marks). The figure below shows isobaric (constant pressure) equilibrium states for water, using the specific volume and the Celsius temperature as thermodynamic coordinates. These states are near the well-known anomalous point for water where the density is a maximum with respect to changes in the temperature. It is evident that if we try to use the pressure P and the volume V as the state coordinates in this region, then two values of the temperature will be obtained for a given pair ( P, V ). Does this situation then imply that the temperature is not a function of state ... contrary to the zeroth law? Explain.

Solution The zeroth law allows us to show formally that the temperature is a property of the thermodynamic state of a system. This principle was recognized by Maxwell (1872) who referred to it as the law of equal temperatures. The modern name was adopted much later in response to a suggestion by Fowler and Guggenheim (1939) who offered the view that This postulate of the existence of temperature could with advantage be known as the zeroth law of thermodynamics. The statement of the law is If the bodies B1 and B2 are in mutual thermal equilibrium, and the bodies B2 and B3 are in thermal equilibrium as well, then bodies B1 and B2 will also be in thermal equilibrium. The figure above appears to cast doubt about the zeroth law and the statement that the temperature is a function of state as this system can have two temperatures while in the same specific volume state. However, the problem is not with the zeroth law but with our interpretation of it.

While it may seem to be improper to enlarge upon the interpretation of laws when inconsistencies arise, physical laws should be discarded only when there is no avenue to resolve them. In effect the focus of this problem moves from the law itself to an understanding of the objects to which it applies. Here, the difficulty occurs because the coordinates ( P, V ) does not constitute a proper pair of state variables. Usually the coordinates ( P, V ) are satisfactory but they cannot be used in all situations, although it is commonly assumed otherwise. The selection of a consistent set of thermodynamic variables is rather like the choice of coordinates for a point in Euclidean space. For example, we can use Cartesian coordinates ( x, y, z ) or we can use spherical coordinates ( , , ) to represent a given point. The choice depends on the unique application, such as the geometric symmetry of the problem of interest. However, points in Euclidean space cannot be represented, in general, by an arbitrary selection of three coordinates chosen from these six. For example, using the coordinates ( , x, y ) would normally identify just two points. Similarly, a consistent set of thermodynamic coordinates for a system cannot be established by selecting variables from a menu in an arbitrary manner. Thus, the state of a fluid cannot always be represented by ( P, V ) and that other coordinates, such as ( V, T ), ( P, T ) or ( U, V ) should be used instead. 9. (10 marks). A body is cooled from an initial temperature, Ta, to a final temperature, Tb. The mass of the body is m and the specific heat of the body for this process is a constant, denoted by c. The temperature of the heat sink for the refrigerator (involved in the cooling process) is also Ta. Show that the minimum work input to the refrigerator for this process is given by . Solution This problem is discussed in Callen (1985), Section 4-5. As stated there the maximum work theorem states that for all processes leading from the specific initial state to the specified final state of the system, the delivery of work is maximum and the delivery of heat is a minimum for a reversible process. The applicable equation is (4.13) which reads where, for our particular case, we have ,

since

10. (10 marks). Two identical bodies, denoted as Bx and By, both with mass m are specific heat c are at temperatures Tx and Ty, respectively. The bodies are located within an adiabatic enclosure. At a particular moment in time, the bodies are placed in thermal contact.

(a) Determine the final, common temperature, Tw, of the composite system Bx + By, and (b) Show that the change in entropy of the composite system is given by . Solution We have two identical systems whose heat capacity CV = mc is independent of temperature. If the systems are brought into thermal contact through a rigid, diathermal wall and isolated from the surroundings, then heat flows from the high T system to the low T system until thermodynamic equilibrium is reached at a temperature Tf. We know from experience that such a process is irreversible and we can calculate the total energy change of the two systems and show that it is positive to confirm this conclusion. The final temperature is determined from the first law of thermodynamics as

whereupon

Since we can calculate changes in entropy in reversible processes we have to construct a reversible processes which takes both systems from their initial state to their final state reversibly. When the heat capacity CV doe s not depend on T, the change in the entropy of the system in a reversible, isochoric process is given by

where the second integral vanishes because the systems volumes do not change and the final integral has been evaluated assuming the specific heat is constant. Thus, using this result for our two bodies

and, since

always, the inequality

hol

expectation of the second law of thermodynamics. Thus, the process of heat flow from a system at higher T to a system at lower T is irreversible. 11. (20 marks). The fundamental relation for a photon gas is given by the expression (1) where c is the speed of light in a vacuum and is the Stefan-Boltzmann constant. (a) Obtain the Helmholtz free energy F ( T, V ) where F is defined as F = U - TS. (b) Show that U = U ( S, V ) can be obtained from the Helmholtz free energy by taking the inverse Legendre transform. (c) Obtain an expression for the pressure, P, as a function of the entropy S and the volume V and as a function of the temperature T and the volume V. That is, use the two definitions of energy U and

F to obtain the pressure. Solution (a) We seek to eliminate the entropy S from the Helmholtz potential definition F = U TS and make the temperature T the independent variable. First, we obtain the temperature as (2) so that we can express the entropy in terms of the temperature as . Hence, and . (3) (4a.b)

Thus, the Helmholtz potential can be written as (5) .

Solution (b) Notice that the expressions for S and U as functions of T and V are not fundamental relations because, taken alone, they cannot be used in general to reconstruct the relation U ( S, V ). However, the Helmholtz relation F ( T, V ) is a fundamental relation, as we can illustrate by taking the inverse Legendre transform. Begin by calculating S using . (6) Hence, we can express T as a function of S, yielding Eq (2) again. Next, by eliminating T from the definition U = F + TS, we finally recover the original fundamental equation, Eq. (1) as required. Solution (c) We can determine the pressure P in two ways. First, as a function of S and V (7) Please notice that ; instead, from Eq (4b), we get . This illustrates the importance of clearly indicating the fixed variables when taking partial derivatives, unless they are really obvious. However, we can obtain P as a function of T and V by using Eq (3) to eliminate S. Alternatively, we can simply take the derivative of F with respect to V using Eq (5) so that (8) and in this form we see that P is a function of T alone. 12. (10 marks). Show that the two energies U and F are related by

. Solution We use two facts to demonstrate this relation. First, the definition of the Helmholtz potenial F = U - TS permits us to write U = F + TS. Next, we recall from the form for dF that

which permits us to identify the definition of S from the Helmholtz potential as . Using this definition for S in the relation U = F + TS yields the desired relation above. This is one of several relations that are collectively known as Gibbs-Helmholtz equations. 13. (10 marks). When the volume of a simple fluid changes in a reversible, adiabatic process the associated temperature change is determined by the derivative , assuming that N is a constant. Obtain an expression for this derivative in terms of the quantities cV, and T. Solution There are two approaches available. In both cases one must bring the entropy S inside the partial derivative. Thus, one possibility is to recognize the Maxwell relation

which arises from the internal energy relation dU = T dS - P dV + dN. The next step is to divide both numerator and denominator by dT to obtain

which simplifies after using the definitions triplet relation

. Alternatively, one may proceed directly using the triplet relation

and then re-arrange as

using the Helmholtz potential dF = - S dT - P dV + dN to alter the numerator. This relation is as derived perviously. The relation also appears reasonable because we expect that an increase in

volume under isentropic conditions (i.e. adiabatic boundaries) would produce a temperature drop. 14. (10 marks). When a simple fluid undergoes an adiabatic throttling process the enthalpy H is constant. The temperature change associated with this type of process is represented by the JouleThomson coefficient which is defined as

where H = U + PV and the mole number N is assumed fixed. Obtain an expression for . Solution The procedure for simplifying the Joule-Thomson coefficient is to first bring the enthalpy within the partial derivative using the triplet relation

so that

after using the expression dH = T dS + V dP + dN for the enthalpy. The denominator simplifies to an expression involving cP since the second partial vanishes while the numerator simplifies after using the Maxwell relation

. As a con

for the Joule-Thomson coefficient. This relation shows that the sign can change depending on the value of the bracketed term. For an ideal gas this term is zero since always.

S-ar putea să vă placă și